LSAT 3 – Section 2 – Question 12

You need a full course to see this video. Enroll now and get started in less than a minute.

Target time: 1:47

This is question data from the 7Sage LSAT Scorer. You can score your LSATs, track your results, and analyze your performance with pretty charts and vital statistics - all with a Free Account ← sign up in less than 10 seconds

Question
QuickView
Type Tags Answer
Choices
Curve Question
Difficulty
Psg/Game/S
Difficulty
Explanation
PT3 S2 Q12
+LR
Sufficient assumption +SA
A
8%
156
B
3%
153
C
20%
158
D
59%
165
E
10%
154
150
158
167
+Harder 148.13 +SubsectionMedium
This page shows a recording of a live class. We're working hard to create our standard, concise explanation videos for the questions in this PrepTest. Thank you for your patience!

This is a sufficient assumption question because the question stem says: “conclusion is properly drawn… which one is assumed?”

Sufficient assumption questions tend to be very formal. We’re looking for a rule that would 100% validate the conclusion, specifically by bridging the premise and conclusion through the rule. Not only are we extrapolating the rule from our argument, but we’re also using that rule to render the argument “valid.” The way to prephrase our answer choice is by tying our premises and conclusion together into a rule: “If [premise] → then [conclusion].” Sometimes though, rules are a little too chunky and don’t capture the gap accurately. This question is a great example of why.

The first sentence is pretty straightforward: photovoltaic power plants (PVP) create electricity through sunlight. We’re also told that it’s cheaper than it was 20 years ago and included the costs of construction and operation in this statement. The passage then notes that corresponding costs have increased for traditional power plants (TP). Okay, all good so far – the author is noting the change of costs for each separately.

The conclusion is that PVP is less expensive than TP. Wait – how can we draw a comparison between the two? We only know what’s going on with their respective costs. We can’t make a comparison between the two when we only know what’s going on within the two respectively. Let’s walk through this:

Imagine in 2003 the cost for PVP was $500. Since then, it’s decreased by 1/10, so the cost today is $50.

Now, for TP, in order for our conclusion to be true, the cost of TP in 2023 would have to be above PVP’s costs in 2023. So, something like: in 2003, the cost for TP was $600, and now the cost is $6000. (A note that “increasing” is a very vague term on the LSAT – we can’t really tell by how much something increased.)

But – does that have to be the case? NO!! It’s totally possible that the cost for TP was $20 in 2003, and then increased to $45. This goes against the conclusion. The problem here is that the premises are only giving information for comparison within the group, and our conclusion is about what’s going on between the groups. There is a mismatch.

If you’re having trouble seeing the comparison issue, let’s take another context. Let’s use the LSAT score as an example. What this argument is saying is the following... I took the LSAT twice and my score went up between those two takes. My friend also took the LSAT but her score went down from her first take. Therefore, I score higher on my LSAT than my friend did.

Is this conclusion valid? Not really - it could be that I went from a 150 to a 154; but my friend went from a 160 to a 159. This is the issue: having information about what’s going on with me and separately with my friend doesn’t allow for a conclusion comparing my score with my friend’s score.

Back to our argument, to close the gap here, a rule feels a little complicated. We need something explicit to address this gap. Something that draws a comparison between the two groups, not just within them. Something like, even 20 years ago, the cost for PVP was way lower than TP plants.

Answer Choice (A) This answer choice is just a repetition of what was stated in the stimulus above. A repetition of our stimulus isn’t going to help us out here.

Answer Choice (B) This isn’t correct – we don’t care about electricity produced 20 years ago. The technology could have improved and PVP produces much more; on top of this, the conclusion is about the cost of electricity now.

Answer Choice (C) Plugging this back into our stimulus will show how this is completely useless to our argument. This answer choice is basically saying that PVP and TP technology works in different ways; so, what? It’s possible to have two ways of doing something and both of them are great. This doesn’t make our conclusion valid.

Correct Answer Choice (D) We’re saying that even back then, PVP was 10 times less than TP. If PVP costs went down by a tenth and we also know that TP costs have increased, then we can properly draw our conclusion.

Answer Choice (E) Once again, we have more information about what’s going in within the groups and no information on what these costs are relative to each other.

 

Take PrepTest

Review Results

Leave a Reply